Jump to content

Probability


Dr Finlay

Recommended Posts

I decided to go over the probability section in my S1 statistics book. However, it seems i've stumbled at the first hurdle. The first exercise is about the addition rule P(AUB) = P(A) + P(B) - P(AnB). There is a question where it asks about the probabilities events not happening, P(A'), P(A'UB), P(A'nB') etc. I know that P(A') = 1 - P(A), but i'm completely stuck on how to get such probabilities of P(A'nB).

 

Can anyone help unclog my mental block?

 

Thanks once again,

Rob

Link to comment
Share on other sites

[Math] P(A\cap B) [/Math]

refers to the probability of the event that is favourable to both conditions=P(A and B)

Say, A refers to an odd no. and B refers to prime no.

A=(1,3,5) and B=(2,3,5)

Then, for a dice, favourable outcomes for that P are (3,5)

Link to comment
Share on other sites

I managed to get that far, but here's an example of a question.

 

If S and T are two events and P(T) = 0.4 and P(SnT) = 0.15 and P(S' n T') = 0.5, find:

 

a) P(S n T')

b) P(S)

c) P(S u T)

d) P(S' n T)

e) P(S' u T')

 

All these questions should be able to be solved using the addition rule. But I can't figure out how to get probabilities of events not occurring from events that do occur, such as a), d) and e).

Link to comment
Share on other sites

Just done another question, i get a different answer from the book though.

 

If A and B are two events and P(A) = 0.6 and P(B) = 0.3 and P(AuB) = 0.8 find:-

a) P(AnB)

b)P(A'nB)

c)P(AnB')

 

For part a) i got 0.1 using the addition rule, which the book says is correct. But for part b) i did

P(A'nB) = (1 - P(A))xP(B)

= 0.4 x 0.3

= 0.12

 

the book however says the answer is 0.2.

 

For part c) i did:

 

P(AnB') = P(A) x (1 - P(B))

= 0.6 x 0.7

= 0.42

 

the book gives the answer 0.5

 

Can anyone see what I'm doing wrong?

 

Thanks alot,

Rob

Link to comment
Share on other sites

Hmm, i think i may have worked it out. I drew a Venn diagram and got

 

[math]P(A \cap B) = P(B) - P(A \cap B)[/math]

 

so it follows

 

[math]P(A \cap B') = P(A) - P(A \cap B)[/math]

 

i then get the book's answers.

I think i just needed to spend the time to work it out, cheers anyway.

Link to comment
Share on other sites

Hmm' date=' i think i may have worked it out. I drew a Venn diagram and got

 

[math']P(A \cap B) = P(B) - P(A \cap B)[/math]

 

so it follows

 

[math]P(A \cap B') = P(A) - P(A \cap B)[/math]

 

i then get the book's answers.

I think i just needed to spend the time to work it out, cheers anyway.

 

[math]P(A' \cap B) = P(B) - P(A \cap B)[/math]

Link to comment
Share on other sites

Create an account or sign in to comment

You need to be a member in order to leave a comment

Create an account

Sign up for a new account in our community. It's easy!

Register a new account

Sign in

Already have an account? Sign in here.

Sign In Now
×
×
  • Create New...

Important Information

We have placed cookies on your device to help make this website better. You can adjust your cookie settings, otherwise we'll assume you're okay to continue.